Definite integral of step function

Click For Summary
The discussion revolves around proving the expression ∑(k=1 to n) ∫(k-1 to k) [x] dx = n(n-1)/2, where [x] is the floor function. The confusion arises from the discontinuity of the floor function at integer points, leading to questions about the validity of the definite integral. It is clarified that definite integrals can exist for functions with jump discontinuities, as the area under the curve remains unaffected by these points. The integral can be computed by recognizing that the floor function is piecewise constant over the intervals, allowing for the correct evaluation despite discontinuities. Ultimately, the expression is affirmed as true, highlighting the importance of understanding the nature of the function being integrated.
Byeonggon Lee
Messages
14
Reaction score
2
I need to prove whether this expression is true or false:

##
\sum\limits_{k=1}^{n}\int_{k-1}^{k}[x]dx = \frac{n(n-1)}{2}
##

I'm so confused because as I know, definite integral is possible only when the target function is continuous in closed interval.

In this case, function ##[x]## should be continuous in interval ##[k-1,k]##
but actually [x] is not continuous in ##[k,k-1]##
##[x]=k-1## when ##k-1\leq x<k##, ##[x]=k## when ##x=k##

So I thought that this expression is false,
but in my book's answer, it is true:

"""
Because##[x]=k-1## when ##k-1\leq x<k##, ##[x]=k## when ##x=k##
##
\sum\limits_{k=1}^{n}\int_{k-1}^{k}[x]dx
##
##
=\int_{0}^{1}[x]dx+\int_{1}^{2}[x]dx+\int_{2}^{3}[x]dx+...+\int_{n-1}^{n}[x]dx
##
##
=\int_{0}^{1}0dx+\int_{1}^{2}1dx+\int_{2}^{3}2dx+...+\int_{n-1}^{n}(n-1)dx
##
##
=0+\bigg[x\bigg]_{1}^{2}+\bigg[2x\bigg]_{2}^{3}+...+\bigg[(n-1)x\bigg]_{n-1}^{n}
##
##
=0+1+2+...+(n-1)
##
##
=\frac{n(n-1)}{2}
##
"""

The book also says that I need to think interval as ##k\leq x<k+1##
and I don't understand how is it possible to omit equal sign at ##x<k+1##
 
Physics news on Phys.org
An integral is an area. The troublesome points at the end of the interval mean you are uncertain about the area at a point. What is the difference in the area under a curve if you move one single point up or down by 1?

Another way to approach it is this. Think of the integral as a limit of a sum of areas. It happens that in this case you can very easily construct the limit because the function you are looking at is piece-wise constant.
 
Byeonggon Lee said:
I need to prove whether this expression is true or false:

##
\sum\limits_{k=1}^{n}\int_{k-1}^{k}[x]dx = \frac{n(n-1)}{2}
##

I'm so confused because as I know, definite integral is possible only when the target function is continuous in closed interval.

The book also says that I need to think interval as ##k\leq x<k+1##
and I don't understand how is it possible to omit equal sign at ##x<k+1##

A definite integral certainly can exist for a discontinuous function. Jump discontinuities in f(x) at an interval's endpoint a or b does not affect the area under the curve y = f(x) from x = a to x = b. In other words,
\int_{[a,b]} f(x) \, dx = \int_{(a,b]} f(x) \, dx = \int_{[a,b)} f(x) \, dx = \int_{(a,b)} f(x) \, dx
We can denote all four of these by the common symbol ##\int_a^b f(x) \, dx##.
 
You can read a little more on this idea at https://en.wikipedia.org/?title=Riemann_integral.
For example, to your point on continuity:
"A function on a compact interval [a, b] is Riemann integrable if and only if it is bounded and continuous almost everywhere (the set of its points of discontinuity has measure zero, in the sense of Lebesgue measure)."
Set of measure zero means a finite (or countably infinite) set of distinct points. In the case of the step function is discontinuous at the integers which for a maximum n is finite, and without a maximum is countably infinite. Both sets are measure zero, so you are okay to integrate.
 
Just to nitpick, or to add a bit: while in this context of step functions discontinuities are finite, you may have in other cases uncountably-infinite sets of measure zero, e.g., the Cantor set.
 
  • Like
Likes RUber
Byeonggon Lee said:
...

The book also says that I need to think interval as ##k\leq x<k+1##
and I don't understand how is it possible to omit equal sign at ##x<k+1##
I believe your function is also called the floor function, ##\displaystyle \ \lfloor x\rfloor \,,\ ## and the greatest integer function, ##\displaystyle \ [\![ x]\!]\ .\ ##

On the interval ##\displaystyle \ [k-1\,,\ k)\,,\ ## we have ##\displaystyle \ [x]=k-1\ .##

Take the limit: ##\displaystyle \ \lim_{a\to k^-} \int_{k-1}^a [x]\,dx \ . \ ##
 
Question: A clock's minute hand has length 4 and its hour hand has length 3. What is the distance between the tips at the moment when it is increasing most rapidly?(Putnam Exam Question) Answer: Making assumption that both the hands moves at constant angular velocities, the answer is ## \sqrt{7} .## But don't you think this assumption is somewhat doubtful and wrong?

Similar threads

  • · Replies 14 ·
Replies
14
Views
2K
  • · Replies 4 ·
Replies
4
Views
1K
  • · Replies 6 ·
Replies
6
Views
2K
Replies
4
Views
2K
  • · Replies 15 ·
Replies
15
Views
2K
  • · Replies 4 ·
Replies
4
Views
2K
  • · Replies 8 ·
Replies
8
Views
2K
  • · Replies 13 ·
Replies
13
Views
2K
  • · Replies 14 ·
Replies
14
Views
3K
Replies
4
Views
2K